Missed MBE - Contracts Flashcards

You may prefer our related Brainscape-certified flashcards:
1
Q

On May 10, the coach of a youth-league baseball team sent a letter to a supplier asking the supplier to promptly ship 20 red jerseys to the coach. On May 15, the supplier received this letter and sent the coach a reply letter accepting the offer. On May 16, the supplier realized that he had no red jerseys with which to fill the order and sent the coach 20 blue jerseys with a note that the blue jerseys were tendered as an accommodation. The coach received the jerseys and accommodation note on May 18 and received the supplier’s acceptance letter on May 19.
On May 20, which of the following is a correct statement of the parties’ legal rights and duties?

A The coach can either accept or reject the blue jerseys and, in either event, recover damages, if any, for breach of contract.
B The coach can either accept or reject the blue jerseys, but if he rejects them, he will thereby waive any remedy for breach of contract.
C The supplier’s shipment of nonconforming goods constituted an acceptance of the coach’s offer, thereby creating a contract for the sale of the blue jerseys.
D The supplier’s shipment of the blue jerseys constituted a counteroffer.

A

A The coach can either accept or reject the blue jerseys and, in either event, recover damages, if any, for breach of contract.

Contracts for the sale of goods (here, jerseys) are governed by Article 2 of the Uniform Commercial Code (UCC). Under the UCC, an offer to buy goods for prompt shipment can be accepted by:
* promising to ship the goods
* shipping goods that conform to the order or
* shipping nonconforming goods without notice that they are being offered only as an accommodation (which also operates as a breach)
Under the mailbox rule, acceptance by mail is effective upon dispatch—not upon receipt. Therefore, the coach’s offer to buy 20 red jerseys was accepted by the supplier on May 15, when he sent the coach a reply letter accepting the offer, thereby promising to ship the goods. This created a valid contract that required the supplier to then deliver goods that conformed perfectly to the contract (“perfect-tender rule”).
Here, however, the supplier breached the contract by shipping blue jerseys instead of red jerseys. As a result, the coach may accept or reject the nonconforming goods in whole or in part, and he may sue for damages related to the supplier’s breach.
(Choice B) A buyer does not waive any remedy for breach of contract by rejecting nonconforming goods. Instead, upon proper rejection, the buyer is entitled to a return of any payments made on the goods.
(Choice C) The coach’s offer was accepted when the supplier dispatched an acceptance letter on May 15—not when the supplier shipped blue jerseys on May 16.
(Choice D) If a seller ships nonconforming goods and seasonably notifies the buyer that the goods are tendered as an accommodation, then the accommodation operates as a counteroffer—unless the parties formed an enforceable contract before the accommodation was sent (as seen here).
Note to self: This is not a merchant agreement, and since there was an agreement via the mailbox rule. Can (1) accept shipment (2) reject shipment (3) accept some, reject others and collect damages. Bottom line: since there was an acceptance on May 15, the nonconforming goods were a breach.

How well did you know this?
1
Not at all
2
3
4
5
Perfectly
2
Q

A maker of handwoven rugs contracted with a supplier to provide yarn made from sheep’s wool. The written contract specified that, for four years, the supplier would provide the rugmaker with 2,000 spools of yarn made from 100% sheep’s wool per month, at $10 per spool, for a total of $20,000. Two years into the contract, the supplier sent the rugmaker 2,000 spools of yarn made from 90% sheep’s wool and 10% synthetic fiber. The rugmaker sent the supplier a check for $15,000 for the shipment, and added a clear note on the check stating that the payment was in full for the shipment but was $5,000 less due to the synthetic fiber in the yarn. The supplier promptly deposited the check, and then four months later filed suit against the rugmaker for the remaining $5,000. The supplier has submitted evidence of the written contract, and the rugmaker has submitted evidence of the deposited check.
What is the rugmaker’s best defense in this situation?
A By depositing the check, the supplier was estopped from claiming that the rugmaker owed him an additional $5,000.
B The rugmaker’s and supplier’s good-faith dispute over the yarn composition suspended the rugmaker’s obligation to pay the remaining $5,000.
C The supplier deposited the check for $5,000 less than the contract price, thereby discharging the rugmaker of any further duty to pay the remaining amount for that month’s shipment.
D The supplier’s act of knowingly depositing the check for $15,000 was a novation that relieved the rugmaker from any further liability.

A

C The supplier deposited the check for $5,000 less than the contract price, thereby discharging the rugmaker of any further duty to pay the remaining amount for that month’s shipment.

Ways to discharge contractual obligations
        * Full performance of contractual obligations
        * Impossibility, impracticability, or frustration of purpose
        * Release (in writing only)
        * Mutual rescission
        * Substituted contract
        * Contract or covenant not to sue
        * Accord & satisfaction
        * Novation
Mnemonic:  FIRM SCAN
An existing contractual obligation can be discharged by an accord agreement.  Under this agreement, a contracting party agrees to accept performance that differs from what was promised in an existing contract in satisfaction of the other party's existing duty.  When a claim is unliquidated or otherwise subject to dispute, it can be discharged by accord and satisfaction if:
	○ the person against whom the claim is asserted tendered a negotiable instrument (e.g., a check)
	○ the instrument was accompanied by a conspicuous statement indicating that it was tendered as full satisfaction of the claim (e.g., "payment in full") and
	○ the claimant obtained payment of the instrument.
Here, the rugmaker sent the supplier a check for $15,000 (instead of $20,000) after receiving yarn made of 90% wool (instead of 100% wool).  The check clearly stated that it was being tendered as payment in full for the supplier's shipment of the lesser-quality yarn.  The supplier then obtained payment by depositing the $15,000 check.  Therefore, the rugmaker's best defense is that there was an accord and satisfaction that discharged the rugmaker of any further duty to pay the remaining $5,000 for the shipment.

	Note to self: Needs a good faith dispute of what is owed. Agreement = accord, payment = satisfaction. Look for (1) good faith dispute (2) payment made to supplier (3) with a conspicuous note that the payment is intended to be in full. Look for supplier party accepting the debt.
How well did you know this?
1
Not at all
2
3
4
5
Perfectly
3
Q

The owner of a restaurant who highlighted local ingredients when creating his menu bought cheese and other dairy products from a local dairy farmer. The owner and the farmer had entered into written requirements contracts each spring for the past 10 years. In the winter of the tenth year, the farmer purchased a substantial amount of new dairy cows and expanded his farming capabilities. He notified all customers that he would have a higher volume and amount of available products the following spring and would adjust deliveries accordingly. The owner responded with a date he wished the products to be delivered, as per custom, but said nothing else. On the agreed-upon date, the farmer delivered substantially more products than he had customarily provided. The owner attempted to accept half of the shipment, as that was roughly his customary quantity, but the farmer stated that the products were already packaged and that the owner should have spoken up after receiving the notice from the farmer. The owner then rejected the shipment in its entirety.
Did the owner breach the contract with the farmer as to this shipment?
A No, because no contract existed, as the parties did not agree to a quantity.
B No, because the farmer made a nonconforming tender of goods.
C Yes, because the owner should have given the farmer time to cure the nonconformity.
D Yes, because the owner rejected the shipment in its entirety.

A

B No, because the farmer made a nonconforming tender of goods.
Under the UCC, a requirements contract is a contract under which the buyer agrees to purchase as many goods as the buyer requires from the seller. And under the perfect-tender rule, the goods and the seller’s tender of those goods must fully conform with the terms of the agreement. Substantial performance will not suffice.
Although goods must generally be tendered in a single delivery, this rule does not apply when the contract or circumstances indicate otherwise. For example, a single delivery would be unreasonable when the buyer would clearly have no room to store the goods if they were delivered all at once. In such an event, the buyer is entitled to reject the delivery for imperfect tender.
Here, the owner and the farmer had a requirements contract. However, the farmer delivered substantially more dairy products than the owner reasonably required, thereby making a nonconforming tender of goods. Considering the owner’s past requirements and the shelf life of dairy products, providing roughly double the amount required in one delivery was clearly unreasonable. Therefore, the owner did not breach the contract when he rejected the farmer’s shipment in its entirety (Choice D).
(Choice A) The quantity term in a requirements contract is sufficiently definite because the quantity can be made certain by reference to objective facts (i.e., the buyer’s actual requirements) or implied in good faith. However, the quantity cannot be unreasonably disproportionate to the stated estimate or prior requirements.
(Choice C) A seller has a right to cure a nonconforming tender if the time for performance has not yet passed. Here, the time for performance was the date of the delivery, so the farmer did not have a right to cure. In any case, the farmer refused to tender a conforming amount when requested by the owner.
Educational Objective:
Under the perfect-tender rule, the goods and the seller’s tender of those goods must fully conform with the terms of the agreement. If the tender of goods in a single delivery would be unreasonable, then the buyer can reject the delivery for imperfect tender.
Note to self: When there are requirement contracts, you want to look at what the individual requires. Any substantial increase/decrease/differentiation is a breach.
Only get to cure a nonconforming tender if it is beyond the date of performance.

How well did you know this?
1
Not at all
2
3
4
5
Perfectly
4
Q

A sister convinced her brother that they should open a small coffee shop. Their friend, a guitarist, suggested bringing his band to play live music in order to attract customers. He did not request any payment, saying that the publicity would be good for the band. The siblings agreed, and the band started playing at the coffee shop weekly. The coffee shop became a success, in no small part due to the band’s performances. When a businessperson offered to buy the coffee shop from the siblings, they orally agreed to each pay the guitarist $10,000 out of their share of the sale proceeds for his help in making the shop popular. The sister told the guitarist about their agreement. He was so delighted with it that he made a down payment on a new car. By the time the sale of the coffee shop was finalized, the brother had encountered financial difficulties. After the sale, the siblings signed a written contract stating that the sister would pay the guitarist $10,000 and her brother would pay him $5,000.
If, after the sale, the brother pays the guitarist only $5,000, will the guitarist have a valid basis for an action against the brother for another $5,000?
A No, because the guitarist was bound by the written modification of the contract made by the siblings.
B No, because the guitarist was only a donee beneficiary of the oral contract between the siblings.
C Yes, because the guitarist’s reliance on the promised payment prevented the siblings from changing the obligations of their oral contract.
D Yes, because the oral promise to pay $10,000 to the guitarist was made binding by the guitarist’s valuable and uncompensated contributions to the business.

A

C Yes, because the guitarist’s reliance on the promised payment prevented the siblings from changing the obligations of their oral contract.

A third-party beneficiary is a nonparty to a contract who receives an advantage or benefit from that contract.  If the contracting parties intended for the contract to benefit the third party, the third party is an intended beneficiary with enforceable rights under the contract.  Those rights vest when the beneficiary:
	○ detrimentally relies on the rights created
	○ manifests assent to the contract at one party's request or
	○ files a lawsuit to enforce the contract.
Once this occurs, the original contracting parties are bound to perform the contract.  Any efforts to rescind or modify the contract after vesting are void unless the third party consents to the rescission or modification.
Here, the guitarist was an intended third-party beneficiary of the siblings' oral agreement to each pay him $10,000.  His rights under that agreement vested when he made a down payment on a car in reliance on the agreement.  This means that the subsequent written modification of that agreement was void because it was made without the guitarist's consent (Choice A).  Therefore, the guitarist may enforce the brother's original promise to pay $10,000.
(Choice B)  An intended beneficiary of a "gift promise" (i.e., a donee beneficiary) may generally sue only the promisor because the promisee is not under an obligation to the intended beneficiary.  But this is of no consequence here since both siblings made a promise for the guitarist's benefit (i.e., both were promisors).
(Choice D)  Something given in the past typically is not adequate consideration to support a contract because it could not have been bargained for, nor could it have been given in reliance upon a promise.  Here, the guitarist already performed at the coffee shop without asking for payment, so this past contribution is not adequate consideration to support the contract.
Educational Objective:
An intended third-party beneficiary is a nonparty to a contract who receives an advantage or benefit from that contract that was intended by the contracting parties.  The beneficiary's right to enforce the contract vests when, for example, the beneficiary detrimentally relies on the rights created.
How well did you know this?
1
Not at all
2
3
4
5
Perfectly
5
Q

A shoe manufacturer contends that the owner of a shoe store called and ordered 50 pairs of Oxford-style dress shoes at $100 per pair to be shipped within three weeks. The manufacturer promptly sent the owner a signed, written acknowledgment of the alleged order that reflected the manufacturer as seller and the owner as buyer, as well as the number and style of shoes. However, the acknowledgment did not indicate the price of the shoes. The owner admits to receiving the acknowledgment the following day and taking no action regarding it. Two weeks later, the owner received a shipment of 50 pairs of Oxford-style dress shoes. The owner immediately called the manufacturer and asserted that he had never ordered the shoes.
Will the statute of frauds prevent the manufacturer from enforcing this contract against the owner?
A No, because an oral contract between merchants is enforceable.
B No, because the owner received and did not respond to the written acknowledgment in a timely manner.
C Yes, because the acknowledgment did not indicate the price of the shoes.
D Yes, because the price of the shoes exceeds the $500 threshold of the statute of frauds.

A

B No, because the owner received and did not respond to the written acknowledgment in a timely manner.

The UCC statute of frauds applies to contracts for the sale of goods valued at $500 or more.  Here, the total price of the shoes exceeds the $500 threshold ($100 × 50 pairs of shoes = $5,000), so the contract must comply with this statute, or meet an exception (Choice D).  To be enforceable, these agreements must be evidenced by a writing that:
	○ provides a reasonable basis to believe a contract was formed (the written acknowledgment)
	○ identifies the parties (the manufacturer as seller and the owner as buyer)
	○ lists the quantity of goods sold (50 pairs of shoes) and
	○ is signed by the party against whom enforcement is sought (the owner).
However, the UCC relaxes the signature requirement in agreements between merchants—i.e., regular dealers of the goods involved in the transaction.  Under the merchant exception, a written confirmation need only be signed by one merchant and sent to the other merchant.  If the recipient has reason to know the confirmation's contents and does not object within 10 days, then it is enforceable against both merchants.
Here, the acknowledgment was not signed by the owner.  But since the owner and the manufacturer are merchants in the sale of shoes, the merchant exception applies.  The acknowledgment meets this exception because it was signed by the manufacturer and received by the owner, who did not object until 14 days after receiving it.  Therefore, the statute of frauds will not prevent the manufacturer from enforcing this contract against the owner.
How well did you know this?
1
Not at all
2
3
4
5
Perfectly
6
Q

A nature magazine advertised a photography contest in its January issue, offering “$1,000 to any subscriber who sends us a photograph of the rare Florida Grasshopper Sparrow that we use for the cover of our May issue. Only submissions meeting our technical specifications and received by April 1 will be considered.” The only subscriber to respond to the advertised contest sent the magazine a photograph of the sparrow that met the magazine’s technical specifications. The photograph arrived on March 15. However, due to an ecological disaster that occurred in early April, the magazine decided to use a different picture on the cover of its May issue. The magazine used the subscriber’s picture on the cover of its June issue and has refused to pay $1,000 to the subscriber on the ground that it was not used on the May cover.
Is the subscriber likely to prevail in a breach-of-contract action against the nature magazine?
A No, because the subscriber’s photograph was not used on the cover of the May issue.
B No, because the subscriber failed to adequately notify the magazine of his acceptance.
C Yes, because all of the express conditions of the offer have been satisfied.
D Yes, because the magazine prevented the publication of the photograph.

A

D Yes, because the magazine prevented the publication of the photograph.

Performance is generally due once a contract is formed, but a duty to perform can be delayed or discharged by a condition—i.e., an uncertain future event that must occur before performance becomes due or is discharged.  There are two types of conditions:
	○ condition precedent – delays performance until a specified event occurs
	○ condition subsequent – excuses performance once a specified event occurs
A condition precedent will be excused if a party whose performance is subject to that condition wrongfully prevents or interferes with its occurrence—e.g., by breaching the duty of good faith and fair dealing (which includes the duty to cooperate) that is implied in every contract.  When this occurs, the condition no longer needs to occur for the interfering party's performance to become due.
Here, the magazine's duty to perform was delayed by three conditions:
	○ receiving a photograph that meets the magazine's technical specifications
	○ receiving the photograph by April 1
	○ using the photograph on the magazine's May cover
The first two conditions were fully satisfied when the subscriber delivered the photograph to the magazine on March 15.  However, the third condition was not met because the magazine failed to cooperate when it chose to use a different picture on the May cover—presumably because the ecological disaster was more newsworthy (Choices A & C).  The magazine's wrongful interference with the occurrence of this condition excused that condition and triggered the magazine's duty to pay the subscriber.  Therefore, the subscriber will likely prevail.
	Note to self: Magazine caused condition to fail. If the condition is in the holder of the party asserting the condition. Here, the Magazine caused the delay, not the subscriber.
		Ask: Is it fair?
How well did you know this?
1
Not at all
2
3
4
5
Perfectly
7
Q

A homeowner called a septic cleaning company and made arrangements for the company to remove the waste from the septic tank on the homeowner’s property. After completing the job, the company mailed the homeowner a bill for $500, the fair market value of the services rendered by the company. The bill indicated that payment was due in 60 days. Upon receiving the bill, the homeowner called the company and informed it that, since he had lost his job due to an accident, he would not be paying the company’s bill. The following day, the company filed suit for breach of contract. Ten days later, the homeowner moved to dismiss the suit. The court granted the motion, dismissing the suit without prejudice.
Is the court’s dismissal proper?
A No, because the parties’ dealings created an implied-in-fact contract.
B No, because the homeowner has repudiated the contract.
C Yes, because the company failed to demand assurances.
D Yes, because the company’s complaint is premature.

A

D Yes, because the company’s complaint is premature.

The doctrine of anticipatory repudiation generally applies when a contracting party clearly and unequivocally indicates an unwillingness to perform a promise before the time for performance is due.  Upon repudiation, the nonrepudiating party may:
	○ treat the repudiation as a breach of the contract or
	○ ignore the repudiation and demand performance.
However, this doctrine does not apply when the date of performance has not passed and the nonrepudiating party has fully performed.  Under those circumstances, the nonrepudiating party must wait until the repudiating party's performance is due before filing suit.
Here, the homeowner clearly and unequivocally stated that he would not pay for the company's septic cleaning services after receiving a bill from the company.  However, the company cannot immediately sue the homeowner because his performance (payment) was not due for 60 days and the company had fully performed.  The company must instead wait until the 60-day period expires before filing suit (Choice B).  Therefore, the company's complaint is premature and the court's dismissal was proper.
How well did you know this?
1
Not at all
2
3
4
5
Perfectly
8
Q

A homeowner entered into oral contracts with both a painter and a landscaper to perform services at his home. The landscaper was the first to begin the services, and shortly after he began to work, he realized that the projected cost of the project would increase dramatically. After the homeowner learned how high the cost of the landscaping services was going to be, he called the painter to tell her that he could not go through with their contract at that time.
The painter stated that she had already purchased a standard set of paintbrushes to paint his home, as well as glass necessary to create a custom mosaic on a back corner of the house, according to the homeowner’s specifications. She had also paid for a temporary city permit to park her utility van on the residential street where the homeowner lived.
In a suit by the painter against the homeowner, which of the following is the painter LEAST likely to recover?
A The contract price minus the market cost of performance.
B The cost of the glass for the mosaic.
C The cost of the paintbrushes.
D The cost of the parking permit.

A

C The cost of the paintbrushes.

After a contract is breached, the nonbreaching party may seek compensatory damages.  These damages primarily aim to put the nonbreaching party in the same position as if the contract had been performed so that the nonbreaching party receives the "benefit of the bargain".  Compensatory damages primarily include all of the following (minus mitigable damages):
	○ expectation damages – the difference between the value of performance without the breach (what was promised) and the value of the performance with the breach (what was received) (Choice A)
	○ consequential damages – losses that arose from the nonbreaching party's special circumstances that were reasonably foreseeable to the breaching party when the contract was made
	○ incidental damages – reimbursement for commercially reasonable expenses that the nonbreaching party incurred as a result of the breach
If such damages cannot be calculated with reasonable certainty, then the nonbreaching party may recover for any expenses incurred in reasonable reliance that the contract would be performed—e.g., the cost of the glass for the mosaic and the parking permit (Choices B & D).*  However, recovery of reliance damages may be reduced by the amount spent by the nonbreaching party on materials that could reasonably be repurposed for another job—e.g., the painter's paintbrushes.  Therefore, the painter is least likely to recover that cost.
*Note that a nonbreaching party cannot recover both expectation and reliance damages, so the painter can only recover the contract price or the price of the glass and the parking permit.
How well did you know this?
1
Not at all
2
3
4
5
Perfectly
9
Q

A jeweler and a goldsmith signed a written agreement that provided as follows: “For $3,000, the goldsmith shall sell to the jeweler a size six gold ring setting that the jeweler shall select from only the goldsmith’s white gold ring designs.” The agreement did not address any other specific terms with regard to the business arrangement between the jeweler and the goldsmith.
When the jeweler arrived to select a ring, he refused to select one of the goldsmith’s white gold ring designs. The jeweler claimed that the goldsmith, immediately prior to the execution of the written agreement, had orally agreed to broaden the jeweler’s choices to also include rose gold ring designs. The jeweler also claimed that the goldsmith had, at the same time, orally agreed to include a set of earring settings, valued at $1,000, as an incentive for the jeweler’s continued business. The goldsmith refused to sell to the jeweler any of his rose gold ring designs or include the earring settings.
If the jeweler sues the goldsmith for damages, how should the court handle the evidence of the alleged oral agreements?
A The court should admit the evidence as to both the promise to include the earring settings and the option to choose a rose gold ring design.
B The court should admit the evidence as to the promise to include the earring settings but not the option to choose a rose gold ring design.
C The court should admit the evidence as to the option to choose a rose gold ring design but not the promise to include the earring settings.
D The court should exclude the evidence as to both the option to choose a rose gold ring design and the promise to include the earring settings.

A

A The court should admit the evidence as to both the promise to include the earring settings and the option to choose a rose gold ring design.

The parol evidence rule generally bars evidence of prior or contemporaneous agreements that contradict the terms of an integrated writing—i.e., a writing that presents the final expression of the parties' agreement.  A writing may be fully or partially integrated.  However, the UCC presumes that a contract for the sale of goods (e.g., jewelry) is only partially integrated.*  As a result, evidence that supplements a written contract is admissible—but evidence that contradicts the writing is inadmissible.
Here, the written agreement between the jeweler and the goldsmith is partially integrated because it represents the parties' final agreement for the sale of a ring—including the price, size, and type of gold.  This writing expressly stated that the ring would be chosen only from the white gold ring designs.  The goldsmith's prior oral statement that he would also include rose gold ring designs contradicts the writing, so the court should not admit this evidence (Choices A & C).
In contrast, the writing made no mention of earrings.  This means that the goldsmith's prior oral statement to include a set of earring settings merely supplements the writing and is not barred by the parol evidence rule (Choices C & D).  As a result, the court should admit the evidence as to the earring settings.
*A contract for the sale of goods will be deemed fully integrated if the court concludes that the parties "certainly" would have included the term in the written contract.  However, this is a difficult standard to meet.
	NOTE TO SELF: Items that are not in the contract may be admitted, but items in the contract cannot admit a contradictory statement.
How well did you know this?
1
Not at all
2
3
4
5
Perfectly
10
Q

A licensing agreement provided that a manufacturer could use an inventor’s patent in manufacturing its products for 10 years. Immediately thereafter, the inventor assigned his rights to receive payments pursuant to the licensing agreement to a corporation. The inventor did not receive compensation for this assignment. The inventor, upon his death five years later, devised his stock in the corporation to his daughter and all of his remaining property to his son.
To whom should the manufacturer make its payments under the licensing agreement?
A The corporation.
B The inventor’s daughter.
C The inventor’s son.
D No one, because the manufacturer’s obligation to make payments under the licensing agreement terminated upon the death of the inventor.

A

C The inventor’s son.

An assignment is the transfer of contractual rights to a third party.  If an assignment is not supported by consideration, then it is a gratuitous assignment and is generally revocable (exceptions listed in the table above).  A revocable assignment is automatically revoked upon the death, incapacity, or bankruptcy of the assignor.
Here, the inventor assigned his right to receive payments under a licensing agreement to the corporation without receiving compensation or other consideration.  Therefore, the assignment was automatically revoked upon the inventor's death, and the right to receive payment returned to his estate (Choice A).  Aside from the stock, the inventor devised all of his property to his son, including the right to receive payment from the manufacturer under the licensing agreement (Choice B).  Therefore, the manufacturer should make payments to the inventor's son.
How well did you know this?
1
Not at all
2
3
4
5
Perfectly
11
Q

The owner of a rare eighteenth-century chest offered to sell it to a connoisseur of antiques for $75,000. The connoisseur countered that she would buy the chest for $50,000. The owner rejected this price. The owner and the connoisseur then executed a written agreement for the sale of the chest at a price to be determined only by a particular antiques dealer whose expertise in valuing this rare item they both trusted. Two weeks later, the agreed-upon antiques dealer examined the chest. He told the owner and the connoisseur that he had to do further research on the chest but that he would let them know his decision in several days. Unfortunately, the dealer died before doing so. A reasonable price for the chest can be established by the court.
Is there likely an enforceable contract?
A No, because the owner and the connoisseur did not intend to be bound unless the dealer set the price of the chest.
B No, because the price of the chest was not determined at the time the agreement was executed.
C Yes, because a reasonable price for the chest can be established by the court.
D Yes, because the owner and the connoisseur executed a written agreement for the sale of the chest.

A

A No, because the owner and the connoisseur did not intend to be bound unless the dealer set the price of the chest.

Under the UCC, a contract for the sale of goods is formed if both parties intend to contract and there is a reasonably certain basis for giving a remedy in the event of a breach. Intent to contract is judged by outward, objective manifestations of intent, as interpreted by a reasonable person. So when an agreement reflects an intent to be bound only if the price is subsequently set, no contract is formed until the price is set.

How well did you know this?
1
Not at all
2
3
4
5
Perfectly
12
Q

A student inherited a large tract of undeveloped land from an eccentric uncle. The student had no present need for the land, and because he had numerous student loans, he decided to sell the land. He advertised a proposed sale of the property, and he was soon contacted by a rancher who owned property adjacent to the offered land. The rancher wanted to purchase the student’s property to expand his ranch and to build facilities for dairy production. The student told the rancher that his car had just broken down and that he was eager to sell the property quickly so that he could repair his car for his commute to class. Although the rancher was fully aware of the fair market value of the property, he offered the student a cash price 80 percent less than the property was worth. The student, disappointed with the low price but desperate to repair his car, accepted the rancher’s offer.
On these facts, which of the following legal concepts would give the student the best chance of canceling the contract with the rancher?
A Bad faith.
B Duress.
C Equitable estoppel.
D Unconscionability.

A

D Unconscionability.
Procedural
Party induced to enter contract without meaningful choice due to deception, compulsion, or significantly unequal bargaining power—eg boilerplate contract provisions that are inconspicuous, hidden, or difficult to understand; contract of adhesion (ie, take-it-or-leave-it contract) when parties have greatly unequal bargaining power
Substantive
Substance of contract itself is duly unfair—eg: one-sided terms; gross disparity in value of consideration exchanged

A court may modify or refuse to enforce a contract on the ground that it is unconscionable—i.e., so unfair to one party that no reasonable person in that party’s position would have agreed to it. Unconscionability can be procedural or substantive, and it is a question of law for the court (not the jury) to decide based on the circumstances. The contract is substantively unconscionable if the terms are unduly unfair.
Here, the student’s best argument for canceling the contract with the rancher is that the contract’s substantive terms are unconscionable. No reasonable person would agree to sell a piece of real property for 80 percent less than it is worth (i.e., for 20 percent of its actual worth). And though there does not appear to be procedural unconscionability, the actual terms of the contract are so unfair that the court could refuse to enforce the contract.

How well did you know this?
1
Not at all
2
3
4
5
Perfectly
13
Q

An honest dispute developed between a condominium owner and a plumber over whether plumbing installed in the kitchen and bathrooms of the condominium satisfied contractual specifications. If the plumbing met those specifications, the condominium owner would owe the plumber $15,000 under the terms of the contract. The condominium owner offered to pay the plumber $10,000 in satisfaction of the owner’s contractual obligations if the plumber replaced the plumbing in the kitchen with another grade of pipe. The plumber accepted the condominium owner’s offer. After the plumber replaced the kitchen plumbing, the condominium owner refused to pay the plumber.
In a breach-of-contract action brought by the plumber, the fact finder determined that the plumbing originally installed by the plumber did satisfy the contract specifications. The fact finder also determined that the plumber and the condominium owner entered into a substitute agreement under which the owner failed to deliver the required performance.
What is the maximum amount that the plumber can recover in damages from the condominium owner?
A $25,000.
B $15,000.
C $10,000.
D Nothing.

A

C $10,000.

Parties to a contract may agree to change one or both parties' performance through an accord agreement or a substitute contract:
	○ Accord agreement – when a party agrees to accept different performance in satisfaction of (i.e., in place of) the original promise; after breach, the party can sue under either the original contract or the accord agreement.
	○ Substitute contract – when the parties form a second agreement that immediately discharges the original contract; after breach, a party can sue under the substitute contract only.
Whether the parties formed an accord agreement or a substitute contract is a fact issue that depends on the formality of the agreement.  The more formal the agreement (e.g., words discharging original duties, consideration on both sides), the more likely the fact finder will determine that the parties intended to create a substitute contract.
Here, the fact finder determined that the plumber and the condominium owner entered into a substitute agreement.  This discharged the original contract for $15,000, so the plumber may sue only under the substitute contract (Choice B).  The substitute contract required that the condominium owner pay $10,000 for the kitchen plumbing, so that is the maximum amount that the plumber can recover (Choice D).
How well did you know this?
1
Not at all
2
3
4
5
Perfectly
14
Q

An independent trucker and a manufacturer entered a written contract for the delivery of a farming implement from the manufacturer to a farmer. Under the terms of the contract, the trucker promised “to deliver a farming implement from the manufacturer to the farmer,” and in exchange, the manufacturer promised “to pay the trucker if the trucker delivers the implement directly to the farmer after picking it up.” The trucker picked up the implement but, instead of driving directly to the farmer, drove 100 miles out of his way to pick up another item from a third party before delivering the implement to the farmer. The manufacturer, unaware that the trucker had failed to deliver the implement directly to the farmer, refused to pay the trucker.
Who has breached this contract?
A Both the trucker and the manufacturer.
B The trucker only.
C The manufacturer only.
D Neither the trucker nor the manufacturer.

A

D Neither the trucker nor the manufacturer.

Note to self: Look at the separate promises made. Here, the issue is that payment was contingent on a direct delivery. However, the trucker’s promise was only to deliver items to the manufacturer.
A breach of contract occurs when a party fails to perform a contractual duty that has become due. Performance may be predicated upon a condition precedent, under which a contracting party’s obligation to perform arises only upon the occurrence of an uncertain future event. If the parties expressly agree to a condition precedent, then the condition precedent will be strictly enforced. This means that a contracting party must fully comply with the condition before the other party’s performance is due.

Here, the trucker fully performed his promise to deliver a farming implement from the manufacturer to the farmer, so the trucker has not breached the contract (Choices A & B).  However, the manufacturer's duty to pay the trucker was expressly predicated on the trucker's direct delivery of the implement to the farmer.  The trucker did not fully satisfy this condition precedent because he took a 100-mile detour, so the manufacturer's performance is not due (Choice C).  Therefore, neither party has breached the contract.
How well did you know this?
1
Not at all
2
3
4
5
Perfectly
15
Q

During the warm months of the year, the owner of a fur coat stored it with the furrier from whom she had bought it. While the coat was at the furrier’s store, a salesperson, mistakenly thinking that the coat was for sale, sold it to a customer. The customer was allowed to reduce the purchase price by the amount of an outstanding debt owed by the furrier to the customer; the customer paid the remainder in cash. In the process of purchasing the coat, the customer was told by the salesperson about the furrier’s storage service but, like the salesperson, was unaware that the coat was not part of the store’s merchandise. After the sale, the owner learned of the transaction between the furrier and the customer. Since the coat had significant sentimental value to the owner, she sought its return from the customer. When the customer refused, the owner filed an action to recover the coat from the customer.
Will the owner likely prevail?
A No, because the customer was a good-faith purchaser of the coat that had been entrusted to the furrier.
B No, because the owner is entitled to damages from the furrier.
C Yes, because the customer did not give full value in acquiring the coat.
D Yes, because the furrier transferred only voidable title in the coat to the customer.

A

A No, because the customer was a good-faith purchaser of the coat that had been entrusted to the furrier.

Under the UCC, which applies to contracts for the sale of goods, the entrustment of goods by the owner to someone who sells goods of that kind (i.e., a merchant) gives the merchant the power to convey good title. Good title can be conveyed to a buyer in the ordinary course of business—i.e., someone who buys goods:
○ in good faith
○ without knowledge that the sale violates the owner’s rights to the goods and
○ from a merchant in the business of selling goods of that kind.
Entrustment includes any delivery and acquiescence to the possession of goods, regardless of conditions expressed between the parties.
Here, the owner stored her coat with, and thereby entrusted the coat to, the furrier—a merchant in the business of selling fur coats. The customer then purchased the coat in good faith and without knowledge that it actually belonged to the owner. As such, the customer is a buyer in the ordinary course who took good title (not voidable title) to the coat (Choice D). Therefore, the owner likely will not prevail in an action to recover the coat from the customer.*

How well did you know this?
1
Not at all
2
3
4
5
Perfectly
16
Q

As part of a divorce settlement, an ex-husband purchased an annuity from an insurance company to be paid to his ex-wife so that she would receive a fixed amount quarterly for the duration of her life. Within a week after the purchase, the ex-wife learned that she had a fatal illness, which had not previously manifested itself but had existed for some time. She died two months later, prior to receiving any payments from the annuity.
The ex-husband has filed suit to rescind the annuity contract.
Will the ex-husband be likely to prevail?
A No, because the annuity contract was a third-party beneficiary contract.
B No, because the ex-husband assumed the risk of his ex-wife’s death.
C Yes, because the ex-wife’s death frustrated the purpose of the annuity.
D Yes, because the ex-husband and the insurance company made a mutual mistake as to the ex-wife’s health.

A

B No, because the ex-husband assumed the risk of his ex-wife’s death.

When both parties are mistaken as to an essential element of a contract (i.e., mutual mistake) at the time the contract is formed, the contract is voidable by the adversely affected party if:
○ the mistake relates to a basic assumption of the contract
○ the mistake materially affects the agreed-upon exchange of performances and
○ the adversely affected party did not assume the risk of mistake.
A party assumes the risk of mistake if, at the time the contract is formed, the party is aware that he/she has limited knowledge of the facts and accepts this knowledge as sufficient.
Here, the ex-husband and the insurance company entered into the annuity contract with no knowledge of the ex-wife’s illness. However, the contract cannot be rescinded based on mutual mistake (Choice D). An annuity contract for the duration of someone’s life assumes that the person will die but does not predict when the death will occur. As such, there is an inherent risk of death before the purchase price is recouped.* Therefore, the ex-husband assumed the risk of the ex-wife’s death and is unlikely to prevail.
*Note that there also is an inherent risk that the person lives longer than predicted, impacting the profitability of the annuity. The insurance company assumed that risk when it sold the annuity to the ex-husband.

17
Q

The owner of a ferry boat operated the boat only during daylight hours during the summer months of June, July, and August. On March 1, the owner entered into a written agreement with a man to serve as the captain of the boat for the upcoming season. On May 1, the owner contracted with a woman to serve as the captain of the boat. On May 30, the man was diagnosed with an illness, and the treatment for this illness prevented him from being employed until the following year. On May 31, the owner learned of the man’s illness and told the man not to worry about their contract as he had found someone else to serve as captain of the boat. The woman served as captain of the boat for the summer months of June, July, and August that year.
On September 1, the man sued the owner for damages based on a breach of their contract.
Can the man recover damages based on breach of contract?
A No, because the man was unable to serve as the captain of the boat during the summer months.
B No, because the owner informed the man about the owner’s contract with the woman prior to June 1.
C Yes, because the owner did not inform the man of the owner’s contract with the woman until after the owner learned of the man’s illness.
D Yes, because the owner’s contract with the woman constituted an anticipatory breach of the owner’s contract with the man.

A

A No, because the man was unable to serve as the captain of the boat during the summer months.

In a bilateral contract, the exchange of promises is sufficient consideration to render both promises enforceable.  Failure to perform the promise at the time performance is due constitutes a breach.  A breach can also occur before the time for performance arises or elapses under the doctrine of anticipatory repudiation.  This doctrine applies when a party clearly and unequivocally, by words or acts, indicates an unwillingness to perform his/her contractual duties.
The nonrepudiating party may generally ignore the repudiation and demand performance pursuant to the contract OR treat the repudiation as a breach.  However, the nonrepudiating party cannot recover damages under the contract if that party is in material breach (i.e., cannot substantially perform his/her obligations).  That is because the material breach discharges the other party's duty to perform.*
Here, the parties formed a bilateral contract when the man promised to captain the boat and, in exchange, the owner promised to pay for the service.  The owner then committed an anticipatory breach by contracting with the woman on May 1 (Choices B & D).  However, the man was unable to serve as the boat captain during the summer months because he was diagnosed with an illness on May 30.  This material breach discharged the owner's duty to pay for the man's services, so the man cannot recover breach-of-contract damages.
18
Q

The owner of a retail clothing store regularly displayed for-sale works by local artists on a wall in the store. An art collector who came into the store inquired about purchasing a particular work for display at his home. The two agreed upon a price, but the collector was not ready to commit to purchasing it immediately. Confident that the collector would purchase the work, the owner promised in a signed writing to sell the work to the collector at the agreed-upon price at any time before the end of the month. On the last day of the month, the collector sent the owner a check for the agreed-upon price, which the owner received on the following day.
If the owner returns the collector’s check and refuses to sell the artwork to the collector, which of the following best supports the owner’s position that a contract had not been formed?
A The collector could not accept the owner’s offer by mailing a check.
B The collector’s acceptance of the owner’s offer was not timely.
C The firm-offer rule is not applicable because the collector was not a merchant with respect to the artwork.
D The firm-offer rule is not applicable because the owner was not a merchant with respect to the artwork.

A

B The collector’s acceptance of the owner’s offer was not timely.

Acceptance of a firm or otherwise irrevocable offer is effective only if it is received by the offeror before the offer expires.
Here, the owner’s signed writing that promised to sell the work to the collector was a firm offer that remained open until the end of the month. Although the collector sent a check to accept the offer on the last day of the month, it was not received by the owner until the following day. Therefore, the best support for the owner’s position that no contract was formed is that the collector’s acceptance of the owner’s offer was untimely.

19
Q

On January 5, a buyer and a seller contracted for the delivery of 100 widgets if they could be delivered by February 20. The agreement was made in a writing signed by both parties and provided that the buyer would pay the contract price of $1,000 upon delivery. On February 3, the buyer and the seller orally agreed to postpone delivery until March 1. However, when the widgets arrived on March 1, the buyer refused to accept or pay for the widgets.
If the seller sues the buyer for breach of contract, who is most likely to succeed in the action?
A The buyer, because any modification of the parties’ contract must satisfy the statute of frauds.
B The buyer, because the agreement on February 3 was not supported by consideration.
C The seller, because the contract modification on February 3 was immediately binding on both parties.
D The seller, because the oral agreement on February 3 waived the February 20 delivery date.

A

D The seller, because the oral agreement on February 3 waived the February 20 delivery date.

A contracting party may generally avoid performance if a condition precedent—i.e., an uncertain future event that must occur before performance becomes due—has not occurred. The nonoccurrence of a condition may be excused, however, if the party who would benefit from the condition waives it by words or conduct. And the waiving party cannot retract the waiver once the other party has detrimentally relied on it.

Here, the buyer’s duty to pay under the original contract was conditioned on the seller’s delivery by February 20. However, the buyer waived the original delivery date by orally agreeing on February 3 to postpone delivery to March 1. The seller detrimentally relied on that waiver by delivering the widgets on March 1, so the buyer cannot retract the waiver. Therefore, the seller will likely succeed in this breach-of-contract action.

20
Q

A caterer contracted with a local farmer for the delivery of three dozen fresh local eggs. The contract provided that because the caterer planned to use the eggshells to serve one of her signature dessert recipes, the eggs needed to be a uniform color.
The farmer delivered the caterer 20 white eggs and 16 speckled eggs. The caterer immediately emailed the farmer and informed him that she was rejecting the eggs because she could not use the inconsistent shells to serve her desserts. The caterer also told the farmer that she did not have the ability to refrigerate the eggs or the space to store them for long and that she would wait for his instructions. The caterer stored the eggs on her countertop for a week and had not heard from the farmer. Concerned that the unrefrigerated eggs would soon spoil, the caterer promptly returned the eggs to the farmer. Due to the perishable nature of the eggs, the farmer had to resell the eggs at half the normal price.
If the farmer brings a breach-of-contract claim against the caterer to recover the full contract price of the eggs, will he succeed?
A No, because the caterer behaved appropriately after rightfully rejecting the eggs.
B No, because the caterer had no obligations regarding the nonconforming eggs.
C Yes, because the caterer had a duty to retain the eggs until the farmer retrieved them.
D Yes, because the caterer was required to sell the eggs on the farmer’s behalf.

A

A No, because the caterer behaved appropriately after rightfully rejecting the eggs.

Under the UCC's perfect-tender rule, a seller must deliver goods that conform perfectly to the contract.  A buyer can therefore reject nonconforming goods within a reasonable time after delivery by promptly notifying the seller of the rejection—as the caterer did here.
After rejection, the buyer has an obligation to take reasonable care of any goods in its possession until the seller has had a reasonable amount of time to retrieve them (Choice B).  When, as here, the seller does not retrieve the goods or provide further instructions, the buyer may generally choose to store, reship, or sell the goods on the seller's behalf.  However, the buyer is required to sell the goods on the seller's account if:
	○ the buyer is a merchant—i.e., one who regularly deals in goods of the kind involved or who, by occupation, holds him/herself out as having knowledge or skills unique to the goods involved*
	○ the goods involved are perishable or threaten to speedily decline in value and
	○ the seller has no local agent to whom the goods can be returned.
Here, the eggs are perishable and there is no indication that the seller had a local agent to whom the eggs could be returned.  But since the caterer is not in the business of selling eggs, she was not a merchant required to sell the perishable eggs on the farmer's behalf (Choice D).  As a result, she acted appropriately when she promptly returned the eggs to the farmer after he failed to give her further instructions.
21
Q

A refrigeration-unit manufacturer contracted with a kitchen appliance store to sell and deliver 100 refrigeration units to the store at a price substantially lower than market value. The written and signed contract included the term “F.O.B. kitchen appliance store, on or before March 30.” The shipping company that the manufacturer normally used to deliver its refrigeration units experienced an unforeseen strike at the end of March. As a result, the manufacturer personally delivered the units to the store on April 18. The store suffered no material harm due to the delay. The refrigeration appliance industry generally allows appliance manufacturers a 30-day leeway for any contractually specified time of delivery, unless such leeway is expressly prohibited by the contract.
If the store brings suit against the manufacturer for breach of contract, which of the following facts provides the manufacturer with the strongest defense to the store’s claim?
A The delay was caused by an unforeseeable strike.
B The manufacturer believed that due to the price at which it offered the refrigeration units, the store would accept a late delivery.
C The store suffered no material harm from the delay.
D There is evidence of a trade usage in the refrigeration appliance industry allowing a 30-day leeway for appliance deliveries.

A

D There is evidence of a trade usage in the refrigeration appliance industry allowing a 30-day leeway for appliance deliveries.

Under the UCC parol evidence rule, evidence of a prior or contemporaneous agreement cannot be used to contradict the terms of a final written agreement.  But evidence of the parties' course of performance, course of dealing, or trade usage can be used to explain or supplement those terms—even when the terms appear unambiguous.  Trade usage is any practice or method of dealing in the particular business or industry that is observed with such regularity so as to justify an expectation that it will be observed in the instant case.
Here, the contract stated that delivery was due on or before March 30.  However, the refrigeration appliance industry generally allows appliance manufacturers a 30-day leeway for a contractually specified delivery date unless expressly prohibited by the contract (not seen here).  Therefore, this evidence of trade usage would provide a strong defense against the store's breach-of-contract claim because it shows that the manufacturer was not in breach when it delivered the units on April 18—within 30 days of March 30.
22
Q

A chemistry professor offered to sell her colleague an autographed first edition of a novel for $1,000. The professor provided her colleague with a signed written statement specifying the terms of the offer and stating that the offer would remain open for one week. Two days later, the colleague learned that the professor had sold the book to someone else in their department. The next day, the colleague showed up at the professor’s office with $1,000, asking to purchase the book. The professor apologized, saying that the book had already been sold.
Is the colleague likely to succeed in an action for breach of contract?
A No, because an option contract is not valid unless the offeror is a merchant.
B No, because the colleague learned that the book had been sold before accepting the offer.
C Yes, because the professor did not revoke the offer prior to the colleague’s acceptance.
D Yes, because the offer was contained in a signed writing and thus could not be revoked.

A

B No, because the colleague learned that the book had been sold before accepting the offer.

An offer can generally be revoked by the offeror at any time prior to acceptance.  If the offeree acquires reliable information that the offeror has taken definite action inconsistent with the offer, then the offer is automatically revoked (i.e., constructive revocation).  Once an offer is revoked, it can no longer be accepted and no contract can be formed.
Here, the colleague learned that the professor had sold the book to someone else before the colleague accepted the offer.  The professor's action was inconsistent with—and automatically revoked—the offer because the autographed first edition of the book was a unique item that the professor could not sell to both the colleague and someone else (Choice C).  Since the colleague could not accept the revoked offer, the colleague is not likely to succeed in this breach-of-contract action.
	NOTE TO SELF: firm offer does not require consideration; option contract does require consideration. There was no consideration for this because there was not an offer made until she was aware that another individual had received the book.
23
Q

On October 1, a retail sporting equipment store telephoned a shoe manufacturer and offered to buy a minimum of 50 and a maximum of 100 pairs of running shoes at $40 a pair to be delivered in 60 days. The manufacturer orally accepted the offer and then immediately faxed a signed letter to the store. The letter contained the following language: “This letter confirms our agreement by telephone on October 1 to sell you 50 pairs of running shoes for 60-day delivery.” Sixty days later, the manufacturer delivered 100 pairs of conforming running shoes to the store. However, the store rejected all 100 pairs of shoes because it had found the same shoes from another seller for $30 a pair. Due to lack of demand, the manufacturer cannot resell the 100 shoes without suffering an economic loss.
Can the manufacturer enforce a contract against the store?

A No, because the manufacturer did not state the agreed-upon price term in the faxed letter.
B No, because the store did not sign the faxed letter.
C Yes, for the sale of 50 pairs of shoes, because the manufacturer’s faxed letter stated that quantity term.
D Yes, for the sale of 100 pairs of shoes, because the store wrongfully rejected that quantity of properly delivered and conforming shoes.

A

C Yes, for the sale of 50 pairs of shoes, because the manufacturer’s faxed letter stated that quantity term.

The UCC uses the battle-of-the-forms rule for contract formation.  Under this rule, an acceptance is effective even if it contains new or additional terms—i.e., one that supplements or complements a term in the offer.*  If both parties are merchants, then new terms become part of the contract unless:
	○ the offer expressly limits acceptance to the terms in the offer
	○ the new terms materially alter the original contract or
	○ the offeror objects within a reasonable time after receiving notice of the new terms.
Here, the store offered to purchase 50 to 100 pairs of shoes from the manufacturer.  The manufacturer's acceptance contained a new term for the sale of 50 pairs of shoes.  Since both parties are merchants, the acceptance was effective despite the new term and that term became part of the contract since none of the bulleted exceptions apply.  Therefore, the manufacturer can enforce a contract against the store for 50 pairs of shoes.
24
Q

On April 1, a buyer and a seller executed a written contract for the sale of an antique car for $40,000, delivery on May 1. The contract contains a clause indicating that it is a total integration of the parties’ agreement. As they each signed the contract, the buyer orally reminded the seller that the buyer’s duty to purchase the car was conditioned on his ability to get approval for a loan by April 20 to fund the purchase. The seller orally agreed, though the condition was not noted in the written contract. When the seller contacted the buyer to execute the sale on May 1, he discovered that the buyer had attempted but failed to get a loan and could not afford to purchase the car. The buyer refused to honor the contract.
If the seller sues the buyer for breach of contract, will the court likely admit the evidence of the oral condition regarding the buyer’s approval for a loan?

A No, because the oral agreement contradicts the terms of the written contract.
B No, because the written contract is a complete integration of the agreement between the parties.
C Yes, as proof of a condition precedent to the buyer’s obligation under the contract.
D Yes, because the oral agreement was a distinct and separate contract.

A

C Yes, as proof of a condition precedent to the buyer’s obligation under the contract.

Evidence of prior or contemporaneous oral or written agreement is admissible to establish:
* whether writing is integrated and, if so, completely or partially
* meaning of ambiguous term
* defense to formation or enforcement (eg, fraud, duress, mistake)
* ground for granting or denying remedy (eg, rescission, reformation)
* subsequent contract modifications
condition precedent to effectiveness

25
Q

A manufacturer of T-shirts contracted with a brand-new clothing store to sell the store 1,000 T-shirts per month for a period of two years. The clothing store’s signature color for its clothing was an orange-tinted red color, called coquelicot, which is very difficult to replicate on a consistent basis. The final, written contract specified that any T-shirts that were not coquelicot could be returned, but it was silent with regard to the return of T-shirts for other reasons.
One year into the contract, the store decided to switch to coquelicot-colored baseball caps instead of T-shirts. As a result, the store returned the most recent shipment of coquelicot-colored T-shirts to the manufacturer and demanded a refund. The manufacturer refused to grant the refund, and the store sued the manufacturer for damages.
At trial, the manufacturer introduced the contract, which clearly stated that T-shirts that were not coquelicot could be returned. The store then attempted to introduce evidence that it had returned coquelicot-colored T-shirts to the manufacturer over the past year without objection and received a refund.
Is this evidence admissible?

A No, because evidence regarding the return of the T-shirts violates the parol evidence rule.
B No, because the express term in the contract regarding the return of T-shirts takes precedence over the course of performance.
C Yes, because the evidence can reasonably establish the parties’ course of dealing on this issue.
D Yes, because the evidence is relevant to show that the manufacturer had accepted the return of coquelicot-colored T-shirts in the past.

A

D Yes, because the evidence is relevant to show that the manufacturer had accepted the return of coquelicot-colored T-shirts in the past.

Under the UCC parol evidence rule, which applies to contracts for the sale of goods (e.g., T-shirts), evidence of prior or contemporaneous agreements cannot be used to contradict the terms of a final written agreement.  However, a course of performance can be used to explain or supplement those terms.  A course of performance is a sequence of conduct that is relevant to understanding an agreement between the parties if:
	○ the agreement involved repeated occasions for performance by a party and
	○ the other party accepted performance without objection and with knowledge of the course of performance.
Here, the manufacturer entered into a final written contract with the clothing store to sell 1,000 T-shirts per month for two years.  The contract stated that non-coquelicot T-shirts could be returned but was silent with regard to the return of coquelicot T-shirts.  This means that the contract's terms can be supplemented with evidence that the store had returned coquelicot T-shirts over the past year without objection and received a refund.  Evidence of this course of performance is therefore admissible.
26
Q

A widow offered to sell her small business, together with all of the business’s assets, to a nonprofit organization. The organization accepted, and on June 1, it signed and executed a contract providing for the sale of the business for $25,000 at the end of the month. When the organization’s agent signed the contract, she orally informed the widow that the organization’s duty to purchase the business was conditioned on obtaining approval from a local zoning board to convert the business’s primary office into an affordable-healthcare clinic. A week later, the woman received another offer to purchase her business for $35,000. At the end of the month, seeking to accept the other offer, the widow refused to honor the contract with the organization because it had neglected to request the necessary approval from the zoning board.
The organization sued the widow for breach of contract. The organization presented clear evidence that it had the necessary funds to perform on the contract at the end of the month, and that the zoning board would have routinely approved the organization’s plans for the office.
Is the organization likely to prevail in its action against the widow?

A No, because the express condition of the zoning board’s approval had not occurred by the end of the month.
B No, because the organization’s failure to seek approval from the zoning board was a repudiation of the contract.
C Yes, because the condition of approval by the zoning board has been waived by the organization.
D Yes, because the condition of approval by the zoning board was not included in the written contract.

A

C Yes, because the condition of approval by the zoning board has been waived by the organization.

A party’s obligation to perform may be conditioned on an uncertain future event that must occur before performance becomes due (i.e., a condition precedent). However, a party whose duty is subject to the condition can waive the condition by words or conduct.
Here, the organization’s duty to perform the contract was subject to the condition that it first obtain approval from the local zoning board. Since the organization did not have to perform until this condition occurred, it had the ability to waive the condition. It did so by making no attempt to obtain approval (Choice A). As a result, the organization can likely enforce the contract without satisfying the condition.

27
Q

In January, a local farmer contracted with a chef to sell the chef a specified amount of local organic tomatoes to be delivered on August 1. On June 15, the farmer called the chef to tell him that part of his crop was infested with tomato fruitworms and he was unsure that he would be able to deliver the full amount requested by August 1. The chef told the farmer that it was absolutely essential that he receive those tomatoes on time to make organic tomato sauce for a restaurant scheduled to open in late August. The farmer assured the chef that he would do his very best to save the crop and deliver by August 1.
Does the chef have valid legal grounds to cancel the contract and order tomatoes from another source?
A No, because the farmer did not state unequivocally that he could not deliver the tomatoes on time.
B No, because the farmer still had more than 30 days to deliver the tomatoes.
C Yes, because the farmer committed an anticipatory repudiation of the contract by causing the chef to feel insecure about the farmer’s performance.
D Yes, because the farmer failed to provide adequate assurances to the chef.

A

A No, because the farmer did not state unequivocally that he could not deliver the tomatoes on time.

Anticipatory repudiation occurs when one party to a contract clearly and unequivocally communicates (through words or conduct) to the other party that it will not perform.  The other party can treat the repudiation as a breach and sue immediately. In contrast, mere insecurity about the party's prospective ability to perform is not a repudiation, but it does give the other party the right to demand assurance of performance.  Under the UCC, which governs contracts for the sale of goods (e.g., tomatoes), the demand for assurances must be made in writing.  Failure to provide adequate assurance within a reasonable time—not to exceed 30 days under the UCC—constitutes a breach.
28
Q

A farmer owned a tractor and offered his brother the chance to purchase it. The farmer told the brother that he had to decide whether he wanted to purchase the tractor within “six months of today’s date.” The brother paid the farmer $200 that day to keep the option open. The agreement was reduced to writing, signed by both men, and dated May 15. The farmer died on July 1. On August 15, the brother notified the executor of the farmer’s estate that he wanted to accept the offer to buy the tractor. The executor refused to sell, and the brother filed suit for the enforcement of the contract.
Is the brother likely to prevail?

A No, because at the time of the farmer’s death, the tractor went to his estate.
B No, because the offer terminated on July 1.
C Yes, because the brother made an enforceable contract to buy the tractor on May 15.
D Yes, because the brother paid $200 to keep the option open.

A

D Yes, because the brother paid $200 to keep the option open.

An offer to form a contract generally remains open for the time stated in the offer or, if no time is stated, for a reasonable time.  However, an offer will terminate prematurely if the offeror dies or becomes mentally incapacitated before the offer is accepted—with one exception.  An option contract will not terminate under such circumstances because the offeree gave separate consideration to keep the offer open for a specified period of time.*
Here, the farmer and the brother entered into a valid option contract on May 15 when the brother paid $200 in consideration to keep his option to purchase the tractor open for six months.  This offer was irrevocable for the entire option period (six months) even though the farmer died on July 1 (Choice B).  Since the brother exercised his option within the six-month period (i.e., before November 15), he is likely to prevail in his suit to enforce the option contract.
29
Q

A groom left his bride at the altar on the day of their wedding. The bride could not bear to keep any painful reminders of the occasion, so she offered to sell her wedding dress to one of her bridesmaids for $5,000. The bride stated that the offer would remain open for 30 days. The bridesmaid said that she was interested but would have to think about it.
A week later, the bridesmaid emailed the bride to ask if the price included the custom-made veil that the bride had worn. The bride did not respond to the bridesmaid’s question. Within the 30-day period, the bridesmaid accepted the bride’s initial offer of $5,000 for the wedding dress. In response, the bride stated that the bridesmaid could only buy the wedding dress for $6,000.
Was a contract formed when the bridesmaid accepted the initial offer of $5,000?

A No, because the bride raised the price of the dress to $6,000.
B No, because the bridesmaid’s question acted as a counteroffer and a rejection of the $5,000 offer price.
C Yes, because the bride was required to keep the initial offer open for the 30-day period.
D Yes, because the bridesmaid’s question did not constitute a counteroffer.

A

D Yes, because the bridesmaid’s question did not constitute a counteroffer.

To form a contract, an offer must be accepted before it terminates. If an offer specifies a date on which it will terminate, then the offer will automatically terminate on that date if it has not been accepted. However, the offer can be terminated at an earlier date—unless the offeree gave separate consideration for the independent promise to keep the offer open for a specified time, thereby forming an option contract.
Here, the bride offered to sell her wedding dress to the bridesmaid for $5,000 and stated that her offer would remain open for 30 days. But since the bridesmaid gave no consideration for the option, the bride’s offer could be terminated before the 30-day deadline (Choice C).

An early termination can occur if, for example, the offeree makes a counteroffer, which acts as a rejection of the original offer and creates a new offer.* However, mere indecision and inquiries (e.g., requests for clarification) do not constitute counteroffers. Therefore, the bridesmaid’s email inquiring as to whether the price included the bride’s custom-made veil did not constitute a counteroffer (Choice B). And since the bridesmaid accepted the $5,000 offer within the 30-day period, a valid contract was formed.
*An exception exists for an option holder, who has the right to make counteroffers during the option period without terminating the original offer.

(Choice A) Had the bride revoked her original offer to sell the wedding dress for $5,000 before the offer was accepted, then the bride could have then raised the price to $6,000. But since the bridesmaid timely accepted the original offer, the bride could not subsequently raise the price.

An offer that specifies a date on which it will terminate will automatically terminate on the specified date. But unless consideration was paid to keep this option open, the offer can be terminated at an earlier date—e.g., if the offeree makes a counteroffer.

30
Q

A library contacted a local artist expressing an interest in purchasing a particular one of the artist’s sculptures for display at the library. The library’s agent and the artist executed a written contract that was signed by both parties and provided that the library would purchase the sculpture for $1,000 due upon delivery of the sculpture to the library. Just before they signed the contract, the agent told the artist, “Plan on delivering the sculpture in 10 days, but please remember that the library’s obligation to purchase the sculpture will be conditioned on the approval of the chairperson of the Artistic Patronage Council, as it will be providing the library with the funds for this sale.” The chairperson of the Artistic Patronage Council orally approved the sale the next day. However, 10 days after the contract was executed, the artist decided that he did not want to sell the sculpture.
If the library sues the artist for breach of contract, is the library likely to prevail?

A No, because the library’s agent made an illusory promise.
B No, because there was no mutuality of remedy when the contract was executed.
C Yes, because the agreement was supported by good consideration even though it was conditioned on an uncertain event.
D Yes, because the artist waived any lack of consideration by signing the contract.

A

C Yes, because the agreement was supported by good consideration even though it was conditioned on an uncertain event.

However, a promise based upon a condition that is within the control of the promisor may be illusory if it essentially promises nothing because the promisor can choose whether to honor that promise.
Here, the library promised to pay $1,000 in exchange for the artist’s sculpture, so their contract was supported by valuable consideration. The library’s obligation to purchase the sculpture was contingent upon a condition precedent—the approval of the chairperson of the Artistic Patronage Council. But this condition did not make the contract illusory because there is no indication that the chairperson’s approval was within the control of the library or its agent (Choice A). Therefore, the library is likely to prevail in its breach-of-contract action.
(Choice B) Mutuality of remedy is not a requirement to form a valid contract.
(Choice D) The basic concept of consideration is that there must be something of substance given in exchange for the promise that is to be enforced. Courts generally will not review the adequacy of the consideration supporting a contract, but the requirement of some form of consideration cannot be waived.
Educational objective:
To be enforceable, a contract must generally be supported by valuable consideration—i.e., a bargained-for change in the legal position between the parties. Performance under a contract may be conditioned upon a condition precedent (which delays performance) or a condition subsequent (which excuses performance).

31
Q

A jeweler who specialized in engagement rings assisted a man who was trying to pick out the perfect engagement ring. The man was inexperienced with the various cuts of diamonds and types of ring settings. Over the course of a few weeks, the jeweler and the man looked at all of the ring styles and discussed pricing based on the man’s budget of $5,000. The man finally settled upon a square-cut diamond with a prong setting that was priced at $5,500. The man initially offered the jeweler $4,500 for the ring. While the man and the jeweler were negotiating the price, the jeweler received a phone call regarding a family emergency. The jeweler told the man that he would email him an offer in the evening, and if they could “meet halfway,” the jeweler would sell the ring to the man. The man agreed.
That evening, the jeweler and the man received emails from one another at the same time. The jeweler’s email contained an offer to sell the ring for $5,000, and the man’s email contained an offer to buy the ring for $5,000. Both emails (i) specified the same style of ring that the two parties had discussed earlier that day, (ii) required payment upon receipt of the ring in two weeks, and (iii) were signed with an electronic signature. Based upon their earlier discussions and the jeweler’s email offer to sell the ring to him for $5,000, the man did not look for an engagement ring at any other jewelry store. When the man showed up two weeks later to pick up and pay for the ring, the jeweler denied that they had a binding contract and would not sell the ring.
If the man sues the jeweler for breach of contract, which of the following most persuasively supports the man’s position?

A A sale-of-goods contract does not require that an acceptance be a mirror image of the offer.
B Both parties conveyed an intent to contract with one another through prior negotiations and the simultaneous emails.
C Since the jeweler was the only merchant in the transaction, the jeweler is estopped from denying that the parties’ correspondence created a binding contract.
D The man detrimentally relied upon the jeweler’s offer to “meet halfway” and the email offer to sell the ring to him.

A

B Both parties conveyed an intent to contract with one another through prior negotiations and the simultaneous emails.

Contract formation under the common law requires an offer with definite terms and an acceptance with knowledge of that offer. But these requirements are relaxed by the UCC, which governs contracts for the sale of goods (e.g., a ring). Under the UCC, a contract is formed if the parties intended to contract and there is a reasonably certain basis for giving a remedy. The contract may be made in any manner sufficient to show agreement—even if the moment of its making is undetermined.
Here, it is uncertain whether a contract between the man and the jeweler was formed during prior negotiations when they agreed to “meet halfway” or when they simultaneously exchanged emails presenting similar offers. However, the negotiations and emails both show their intent to contract, and the emails give a clear basis for a remedy against the jeweler. Therefore, this is the strongest argument supporting the man’s position that a valid contract was formed and breached by the jeweler.

32
Q

A man was moving to another state and decided that he wanted to give away some of his belongings. The man knew that his brother had always expressed interest in the man’s antique desk. The man called the brother and said, “I’m going to be moving in two weeks. I would like to give you the antique desk as a gift. I’ll drop it off at your house on my way out of town.” The brother told the man that he was very grateful for the gift and was looking forward to having the desk in his home office. The brother, in reasonable reliance on the man’s promise, immediately disposed of his old desk and made room for the antique one.
A couple of days later, an appraiser, who was a friend of the man, visited the man’s house for dinner. While at his house, the appraiser saw the antique desk and informed the man that it was worth well over $20,000. The man decided to keep the desk and did not drop it off at the brother’s house on his way out of town.
The brother brought suit against the man to recover the antique desk.
If the court finds in favor of the man on these facts, what is the most likely reason?

A A promise to make a gift in the future cannot be enforced.
B The brother did not rely to his detriment on the man’s promise.
C The man’s promise was not in writing.
D The man’s refusal to give the antique desk did not cause injustice.

A

D The man’s refusal to give the antique desk did not cause injustice.

A contract must be supported by consideration—i.e., a benefit bargained for and received by the promisor from a promisee. Since a promise to make a gift does not involve a bargained-for exchange, it is generally unenforceable. But under the doctrine of promissory estoppel, a gift promise is enforceable if three requirements are met:
* the promisor should reasonably expect the promisee to rely on the promise
* the promisee detrimentally relies on the promise and
* injustice can be avoided only by enforcement of the promise.
In this case, the man’s promise to gift the antique desk to his brother will only be enforced if all three of these requirements are met. The facts indicate that the brother did reasonably and detrimentally rely on the promise by disposing of his old desk (Choice B). Therefore, if the court finds in favor of the man, the court must have concluded that the man’s failure to give the antique desk to the brother as promised did not cause injustice.

33
Q

A produce wholesaler sent a written offer to a farmer to purchase all of the corn that the wholesaler required for his business from the farmer for a period of two years. Excited at the prospect of having a guaranteed sale for his corn, the farmer immediately communicated his acceptance to the wholesaler. The wholesaler and the farmer entered into a written contract reflecting the basic terms set forth in the wholesaler’s offer.
Six months after the contract was executed, the wholesaler determined that, while the farmer’s corn was returning a profit, the farmer’s corn was not selling as well as corn that the wholesaler could acquire from other sources. The wholesaler contacted the farmer and informed him that he no longer required any of the farmer’s corn and would not be placing another order. The wholesaler immediately started buying his corn from another source.
If the farmer sues the wholesaler for breach of contract, is he likely to prevail?

A No, because the contract did not contain a specific quantity term.
B No, because the wholesaler no longer needed the farmer’s goods.
C Yes, because the farmer relied on the wholesaler’s promise.
DYes, because the wholesaler purchased corn from another source.

A

D Yes, because the wholesaler purchased corn from another source.

Buyer’s duty of good faith & fair dealing under requirements contract
* Buyer must purchase goods in question from seller only
* Requirements must approximate reasonably foreseeable figure
* Reasonable elasticity in requirements permitted (so long as any variations are in good faith)
Article 2 of the Uniform Commercial Code (UCC) governs contracts for the sale of goods. Under the UCC, a requirements contract is a contract for the sale of as many goods as the buyer requires during a specified period. This creates an exclusive agreement between the buyer and the seller. As a result, the duty of good faith and fair dealing implied in every contract requires the buyer to purchase goods from the seller only. A failure to do so violates that duty and is a breach of contract.

34
Q

Prior to her death, a celebrity commissioned an artist to paint a portrait of her. The celebrity hired this particular artist because he painted using an old-fashioned and rarely used style that required two months of daily appointments during which the subject would sit for a few hours each day. The contract between the parties specified that this live-model method would be used and that the celebrity would deliver increasing payments throughout the process, with the first payment occurring after two weeks of painting. One week into the process, after the painting had begun, the celebrity died. Her family demanded that the artist continue with the painting, using photographs as a substitute for the daily sessions.
Is the artist required to complete a painting of the celebrity?
A No, because no payment had yet occurred.
B No, because the celebrity died after only one week.
C Yes, because the artist can complete the painting by relying on photos of the celebrity.
D Yes, because the artist had already begun painting the celebrity.

A

B No, because the celebrity died after only one week.

A contracting party’s duty to perform is discharged by impracticability when (1) an unanticipated or extraordinary event makes it impracticable for the party to perform, (2) the contract was formed under a basic assumption that the event would not occur, and (3) the party seeking discharge was not at fault in causing the event to occur.

35
Q

A car salesman on the lot at a car dealership specializing in expensive, high-end vehicles called out with a megaphone, “Free set of wheels to the next person who buys a car from me!” He was surrounded by stacks of tires and a display case with informational pamphlets advertising all-weather tires. A customer believed that the car salesman was offering a two-for-one deal on the expensive vehicles sold on the lot. Therefore, the customer immediately approached the salesman and purchased a new vehicle for his son, believing that he would have a nice new car to give to his daughter as well. The salesman then asked if the customer would like his free set of tires installed on the new car. The customer responded that he did not want the tires and that the salesman was contractually obligated to give him a second car. The salesman laughed and said that he was never offering a free car but would cancel the sale of the first car as a consolation.
Is the salesman obligated to give the customer a free car?
A No, because no reasonable person would believe that the salesman was offering a free car.
B No, because the salesman was willing to cancel the sale.
C Yes, because only the customer has the right to avoid or reform the contract for misrepresentation.
D Yes, because the customer honestly believed that the salesman was making a valid offer.

A

A No, because no reasonable person would believe that the salesman was offering a free car.

Contract formation requires a manifestation of mutual assent, which occurs upon acceptance of a valid offer to contract. The validity of an offer and an acceptance is assessed under the objective theory of contracts. This means that a party’s intent to contract is judged by outward objective facts as interpreted by a reasonable person—not a party’s subjective intent or belief (Choice D). If there is no objective “meeting of the minds,” then no contract is formed.
Here, the salesman offered a “free set of wheels” to the next person who bought a car from him. Although “wheels” is a slang term for a car, no reasonable person would believe that the salesman intended to offer a two-for-one deal on the expensive vehicles—especially since he was surrounded by tires and tire pamphlets. Instead, a reasonable person would interpret the offer to mean a free set of tires. Therefore, the salesman is not obligated to give the customer a free car.

36
Q

t the auction of construction equipment owned by a contractor, several lots were offered for bidding and the highest bids for each were accepted by the auctioneer. The auctioneer then announced that a lot that consisted of a backhoe was being auctioned off. Several bids for the backhoe were acknowledged by the auctioneer. Just before the auctioneer brought down her gavel, she glanced at the contractor. The contractor gave the auctioneer a prearranged signal. Acting in accord with the signal, the auctioneer stated that the backhoe was being removed from the auction. There had been no indication as to whether the auction was being held with or without reserve.
The highest bidder on the backhoe, contending that he is now its owner, has brought suit against the contractor.
How is the court likely to rule?
A For the contractor, because the auctioneer had not brought down the gavel, announcing the completion of the sale of the backhoe.
B For the contractor, because the backhoe constituted equipment.
C For the highest bidder, because the contractor forfeited his right to withdraw the backhoe by prearranging a signal with the auctioneer.
D For the highest bidder, because the contractor lost the right to withdraw the backhoe once the auction began.

A

A For the contractor, because the auctioneer had not brought down the gavel, announcing the completion of the sale of the backhoe.

The UCC has special rules for goods sold at auction. If goods are auctioned in lots, each lot represents a separate sale. Whether the goods can be withdrawn once the auctioneer calls for bids depends on the type of auction:
* at a reserve auction—which is presumed unless a no-reserve action is announced—the auctioneer may withdraw goods from auction prior to completion of the sale
* at a no-reserve auction—which must be specifically announced—goods cannot be withdrawn from auction after the auctioneer calls for bids unless no bid is received with a reasonable time
In either type of auction, a bidder may retract a bid until the auctioneer announces the completion of the sale (e.g., at the fall of the auctioneer’s hammer). However, the bidder’s retraction will not revive any earlier bids.
Here, the auction was presumably held with reserve since there is no indication that a no-reserve auction was specifically announced. This means that the auctioneer permissibly withdrew the backhoe upon receiving the contractor’s signal prior to bringing down her gavel, announcing the completion of the sale. As a result, the contractor remains the owner of the backhoe, and the court will likely rule in his favor.

37
Q

A woman sent an offer to sell her office printer to her friend for $450. In her offer, the woman said that the friend was welcome to mail her acceptance to the woman’s business address but that the friend had to let the woman know within the next week whether she was interested. The friend needed an office printer, so she immediately accepted the woman’s offer by mailing a letter to the woman’s home address. Later that same week, thinking that the friend was not interested, the woman sold the office printer to a different person. A few days later, after the one-week deadline had passed, the friend’s letter was delivered to the woman’s house. The woman called the friend thereafter and told her that the office printer had already been sold.
Will the friend likely succeed in an action for breach of contract?

A No, because the offeror determines the manner and means by which an offer may be accepted.
B No, because the woman did not receive the friend’s acceptance letter until after the one-week deadline had passed.
C Yes, because the offer was irrevocable for at least one week.
D Yes, because the woman did not specify that mailing an acceptance to her business address was the only mode of acceptance.

A

D Yes, because the woman did not specify that mailing an acceptance to her business address was the only mode of acceptance.

Contract formation requires both an offer and an acceptance (i.e., mutual assent). The offeror can dictate the manner and means by which the offer may be accepted. But if the offeror does not do so, then the offeree can accept the offer in any reasonable manner and by any reasonable means—e.g., delivering the acceptance by mail. Under the mailbox rule, an acceptance sent by mail or similar means is generally effective upon dispatch, while a rejection is effective upon receipt. Here, the woman did not dictate that an acceptance must be mailed to her business address; she merely welcomed the friend to mail it there. Therefore, the friend could accept the offer by another reasonable means (Choice A). The friend did so by immediately mailing her acceptance to the woman’s home address. Since that acceptance was effective upon dispatch, the friend accepted the woman’s offer before the one-week deadline had passed (Choice B). As a result, the friend will likely succeed in her breach-of-contract action.

38
Q

A butcher and a seller entered into a written contract for the purchase and sale of a building to be used as a butchery. The closing was scheduled for June 1. On May 25, the seller was notified by the city that the building, which had previously been used as a butchery, had a number of significant city code violations. The seller immediately contracted with an electrician and others to correct the issues. Despite his best efforts, the seller realized that the building would not be brought up to code until at least June 10. The seller promptly sent written notification of this issue to the butcher and informed him that he would be unable to take possession of the building until June 10.
Based on his agreement with the seller, the butcher had declined to renew his lease at his current location and was forced to remove his equipment and inventory from that location by the end of May. This included a large freezer that the butcher regularly used to store his inventory. Between June 1 and June 10, to prevent spoilage of his inventory due to the delay, the butcher had to rent space to store his equipment and inventory. He moved his freezer to the rented space to store his meat at a cost of $200 per day, plus the cost of electricity to run the freezer. The cost of electricity to operate the freezer would have been the same no matter where the freezer had been stored.
On June 10, the building was up to code. The butcher paid the seller the agreed-upon purchase price of $300,000 and took possession of the property.
What damages, if any, may the butcher recover from the seller?

A Nothing, because the seller acted in good faith.
B Nothing, because the contract did not contain a “time is of the essence” clause.
C $2,000 for the rental cost the butcher had to pay from June 1 to June 10.
D $2,000 plus the cost of electricity to run the freezer.

A

C $2,000 for the rental cost the butcher had to pay from June 1 to June 10.

A closing date is not an essential term of a real estate contract. Therefore, a seller’s performance is typically due at or within a reasonable time after the closing date—unless the real estate contract contains a “time is of the essence” clause (not seen here). Accordingly, the seller’s failure to perform by the closing date is not a material breach that excuses the buyer’s duty to perform—but it is still a breach. The buyer can therefore recover damages—even if the seller acted in good faith (Choices A & B).
The goal of contract damages is to place the nonbreaching party in the same position as if the contract had been performed (i.e., expectation damages). This loss in value is measured by the difference between the value of performance without the breach (what was promised) and with the breach (what was received). For late delivery in a real estate contract, such damages are measured by the fair market rental value of the property for the time that the buyer (here, the butcher) was denied possession (here, $2,000).
The buyer can also recover other losses, including consequential and incidental damages, minus any costs or losses avoided. Here, the butcher had to run his freezer at the rented space. But he did not incur a loss because he would have had to pay the same amount to run the freezer in the building he purchased had he been able to timely move in. Therefore, he cannot recover the cost of electricity to run the freezer (Choice D). The butcher can only recover the rental cost paid from June 1 to June 10.

39
Q

A father, hoping to build a new playground for his children, invited a friend whose hobby was woodworking to lunch. During the lunch, the two men discussed an arrangement in which the woodworker would build and deliver a swing set to the father for $2,000 within two weeks. Later that day, the woodworker sent an email to the father restating what had been discussed. The father immediately responded in a signed email stating, “We have a deal. But please deliver the set within one week instead.” The woodworker did not respond but began working on the swing set that day.
Eight days later, the father called the woodworker to ask why the swing set had not been delivered. The woodworker stated that he intended to deliver the swing set within the two-week period originally discussed. He began to work more quickly to complete the swing set sooner and delivered the swing set two days early, but the father refused to pay him for it.
Under the UCC, is the woodworker entitled to recover the $2,000?

A No, because the parties did not agree to all essential terms.
B No, because the woodworker did not deliver the swing set within one week.
C Yes, because the goods were specially manufactured.
D Yes, because the woodworker delivered the swing set within two weeks.

A

D Yes, because the woodworker delivered the swing set within two weeks.

The UCC, which governs contracts for the sale of goods (here, a swing set), uses the battle-of-the-forms rule for contract formation. Under this rule, an acceptance that contains new or revised terms is still an acceptance (not a counteroffer) so long as it does not require assent to new or revised terms. And if at least one party is a nonmerchant, then the new or revised terms are merely treated as proposed additions to the contract.
Here, the father’s email accepted the woodworker’s offer but revised the delivery time from two weeks to one week. Since the father did notcondition his acceptance on this revised term, the offer was accepted and a contract was formed. The revised term was merely a proposed addition to the contract because the father is a nonmerchant. The woodworker did not accept the proposed one-week deadline, so he timely delivered the swing set by the original two-week deadline (Choice B). Therefore, he is entitled to $2,000.